Welcome to the first ever Brilliant Polynomial Roots Contest. This is inspired by many other contests in Brilliant. The aim is to improve the skills of Brilliant users in olympiad problems that ask you to find some functions involving the roots of a polynomial by vieta's, newton sums or other methods. The rules are:
I will post the first problem. If someone solves it, he or she can post a solution and then must post a new problem.
A solution must be posted below the thread of the problem. Then, the solver must post a new problem as a separate thread.
Please make a substantial comment.
Make sure you know how to solve your own problem before posting it, in case no one else is able to solve it within 36 hours. Then, you must post the solution and you have the right to post a new problem.
If the one who solves the last problem does not post a new problem in 24 hours, the creator of the previous problem has the right to post another problem.
No restriction in techniques you can use! use of calculus and roots of unity is allowed. use of cyclotomic polynomials and möbius inversion is also allowed.
It is NOT compulsory to post original problems. But make sure it has not been posted on Brilliant.
Your question must have a polynomial. it can be like or . Both are allowed as long as the simplified form is a polynomial.
Format your proofs as
SOLUTION TO PROBLEM n
proof here
Question as
PROBLEM n
ask question relevant question here
To answer the latest question just shift to the "newest" mode
EDIT: for external discussion go to the discussion board
Easy Math Editor
This discussion board is a place to discuss our Daily Challenges and the math and science related to those challenges. Explanations are more than just a solution — they should explain the steps and thinking strategies that you used to obtain the solution. Comments should further the discussion of math and science.
When posting on Brilliant:
*italics*
or_italics_
**bold**
or__bold__
paragraph 1
paragraph 2
[example link](https://brilliant.org)
> This is a quote
\(
...\)
or\[
...\]
to ensure proper formatting.2 \times 3
2^{34}
a_{i-1}
\frac{2}{3}
\sqrt{2}
\sum_{i=1}^3
\sin \theta
\boxed{123}
Comments
PROBLEM 7
If a,b,c are the roots of the equation
x3−2x2−3x−4=0
Show that a−ba2016−b2016+b−cb2016−c2016+c−ac2016−a2016 is an integer.
This problem has been solved by Aareyan Manzoor but after time was finished, so Dev Sharma can Post the next problem
Log in to reply
Simple induction man, it not only holds for 2016 but for all natural no. n in the powers.
PROBLEM 1 given the roots of y8−y7+y6−1=0 are y1,y2,...,y7,y8 find the value of y191+y291+...+y791+y891 This problem has been solved by dev sharma
Log in to reply
SOLUTION TO PROBLEM 1
y8−y7+y6−1=0
Now we have to form a polynomial whose roots are y11,.....
Now, let x=y1
then y=x1
Putting it in the above equation, we get
x8+x2−x+1=0
x8=x−x2−1
x9=x2−x3−x...(1)
Putting x=y11,...,y81 repeatedly in the equation and add them all and using bit Newton Sum (S1=S2=S3=0)
And answer is 0.
Let wk denote the 2015th primitive roots of unity. Find
n=1∑ϕ(2015)wnk=1∏ϕ(2015)wk
This problem has been solved by Aareyan Manzoor
Log in to reply
SOLUTION TO PROBLEM 4 you forgot to format your question properly! the proof anyways:
claim: the product of all the primitive roots of unity are 1.
proof: we know that each primitive root of unity can be written as en2kπi.for k coprime to n. when we multiply all of these we find: exp(n2πi1≤x≤2015,gcd(x,2015)=1∑x) the formula for the sum of all co-primes less then n is 2nϕ(n) view this. put that there to get exp(ϕ(n)πi) ϕ(n) is always even apart from n=1,2. so this will always be 1 for n>2. at 2015 it is one then. the rest is easy i guess. we put 1 to find ∑wn=μ(2015)=−1
PROBLEM 6
Let the roots of p(x)=x3−2x−2 be x1,x2,x3. Evaluate: i=1∑3xi2+xi+1xi+1
This problem has been solved by Dev Sharma
Log in to reply
SOLUTION TO PROBLEM 6
x3−2x−2=0
x3−1=2x+1...(1)
Now, lets see what we need to find,
x2+x+1x+1
Now multiplying x−1 in numerator and denominator.
x3−1x2−1
Now using (1)
2x+1x2−1
Now, using vieta
x1+x2+x3=0
x1x2+x2x3+x3x1=−2
x1x2x3=2
Now putting x=x1,x2,x3 in the expression we want to find out and simplifying it, we get
2x1+1x12+2x2+1x22+2x3+1x32−(2x1+11+2x2+11+2x3+11)
Now using those vieta relations, we can get answer 3−1
Log in to reply
We can simplify our last expression like this -
2x1+1x12+2x2+1x22+2x3+1x32−(2x1+11+2x2+11+2x3+11)
= (2x1+1)(2x2+1)(2x3+1)x12(4x2x3+2x2+2x3+1)+x22(4x1x3+2x1+2x3+1)+x32(4x1x2+2x1+2x2+1) - (2x1+1)(2x2+1)(2x3+1)(4x2x3+2x2+2x3+1)+(4x1x3+2x1+2x3+1)+(4x1x2+2x1+2x2+1)
Now, simplify it and get the answer!!
Log in to reply
PROBLEM 3 find ∑ln(1−ω3) where ω are 2015th roots of unity≠1.
This problem has been solved by Julian Poon
PROBLEM 5 given the polynomial x4+x3−2x+11=0 a,b,c,d are its roots. evaluate cyc∑a4+11a3(a+1)
This problem has been solved by Alan Enrique Ontiveros Salazar
Log in to reply
SOLUTION TO PROBLEM 5
Surely there's an easier way, but this is a new way:
x4=−x3+2x−11x5=x3+2x2−13x+11x6=x3−13x2+13x−11
So x4+11x3(x+1)=2x−x32x−11
Now we say that 2x−11=(2x−x3)(Ax3+Bx2+Cx+D). Expanding and substituting the first three relations we get:
2x−11=(−3A+B+C−D)x3+(13A−2B+2C)x2+(−9A+13B−2C+2D)x−11A−11B+11C
Matching the coefficients and solving the system we get A=152, B=154, C=−53 and D=−1511.
Hence, 2x−x32x−11=152x3+154x2−53x−1511
So we want to find 152∑xi3+154∑xi2−53∑xi−1511(4)
Using Newton's Sums we get S1=−1,S2=0,S3=2,S4=11 and P1=S1=−1, P2=S1P1−2S2=1, P3=S1P2−S2P1+3S3=5.
So the sum is 152(5)+154(1)−53(−1)−1511(4)=−57.
Problem 15
Prove that there doesn't exist polynomials with integer coefficients fk(x)(k=1,2,3,4), such that 9x+4=f1(x)3+f2(x)3+f3(x)3+f4(x)3
solved by Xuming Liang
Log in to reply
Solution: Consider the third root of unity w=1. Since w2=−1−w, we can express fi(w)=aw+b for some integers a,b. Thus fi(w)3=(aw+b)3=3ab(b−a)w+a3+b3−3a2b. Note that the coefficient of w is always even, therefore the coefficient of w on the LHS of the equality is even. However the same coefficient on the RHS is odd, thus these polynomials cannot exist.
Log in to reply
dont you think you need to prove thet fi is linear?
Problem 16
Among the polynomials P(x),Q(x),R(x) with real coefficients at least one has degree two and one has degree three. If P(x)2+Q(x)2=R(x)2 prove that one of the polynomials of degree three has three real roots.
Solved by Khang Nguyen Thanh
Log in to reply
SOLUTION OF PROBLEM 16: This is a problem of All-Russian MO 2002.
Define deg(P) to mean the degree of P(x), and likewise deg(Q) and deg(R).
Observe that max{deg(P),deg(Q)}=deg(R).
This is because deg(R2)=deg(P2+Q2)=max{deg(P2),deg(Q2)} (leading coefficients of P2 and Q2 are positive and do not cancel), and 2deg(R)=deg(R2)=max{deg(P2),deg(Q2)}=2max{deg(P),deg(Q)}.
Thus one of P,Q (without loss of generality, Q) must have degree 2, and both P and R must have degree 3.
Now we write P2=(R+Q).(R−Q). We know that both factors have degree 3.
Since P has either 1 or 3 real roots, R+Q and R−Q either both have one real root or three real roots accordingly.
Assume that P,R+Q and R−Q each have one real root; let the root of P be r1.
Since r12∣P2, this means that r1 is a root of both R+Q and R−Q, and is therefore a root of both R and Q.
Now let P0,Q0, and R0 be the three polynomials P,Q, and R with the common root divided out.
We have P02+Q02=R02.
Then all three polynomials have real coefficients, P0 and R0 have degree 2, and Q0 has degree 1.
We also know that P0 has 0 real roots, Q0 has 1, and R0 has 0 or 2.
Since the latter case would imply that R has 3 real roots, we instead assume that R0 has 0 real roots.
Writing Q02=(R0+P0).(R0−P0), we see that one of the factors must have degree 2 and the other degree 0.
If the first factor has degree 2, we may write P0=ax2+bx+c,R0=ax2+bx+d for real a,b,c,d.
Then Q0=(d−c)(2ax2+2bx+c+d). Since P0 and R0 have no real roots, we know that b2−4ac<0 and b2−4ad<0.
Adding the two inequalities and multiplying by 2, this means 4b2−8a(c+d)<0.
But then Q0 has no real roots, a contradiction.
Similarly, if R0+P0 has degree 0, R0−P0 has degree 2, we write P0=ax2+bx+c,R0=−ax2−bx+d.
Then Q0=−(c+d)(2ax2+2b+(c−d)).
Again, P0 and R0 have no real roots, so b2−4ac<0 and b2+4ad<0.
Adding and multiplying by 2 gives 4b2−8a(c−d)<0, which again implies that Q0 has no real roots.
Thus our original assumption is false, and either P or R must be a third-degree polynomial with 3 real roots.
Clearly, R(x) has degree 3 and P(x) and Q(x) have degree 2 and 3 (W.L.O.G.)
Also,
R(x)=Q(x)±iP(x)
From the above equation, it is clear that if R(x) has a real root, it is also a root of P(x) and Q(x). So, R(x) has at most 2 real roots (counted with multiciplity). (Since P(x) has degree 2 and non real roots occur in conjugate pairs)
If R(x) has 2 real roots (one with multiplicity), then we are done. If not, then let α be the common real root to all three polynomials.
Let β1 and β2 be the other two roots of Q(x),
Since,
Q2(x)=(R(x)+P(x))(R(x)−P(x))
and the two polynomials in the brackets are not identical, the set of roots must be α, β1, β1 and α,β2,β2. Again, since non real roots occur in conjugate pair, we must have β1,β2∈R. Thus Q(x) has three real roots.
PROBLEM 20:
If a,b,c are roots of the polynomial x3−3x2+2x+3=0 , find the value of a3+b31+b3+c31+a3+c31
Easy , because its created by me ;)
This problem is solved by Aareyan Manzoor
Log in to reply
By newtons sum P3=0. so ∑a3+b31=∑−c31 pu x=1/x to have x3+32x2−x+31=0 −∑x3=−((∑(x))((∑x)2−3(∑x1x2))+3∏x) the result follows from vietas.
Yo @Aareyan Manzoor damn you wrote the solution first ,
PROBLEM 2
If 1,x1,x2,.....,x48 are 49th roots of unity, then find
i=1∑48(1−xi)31 This problem has been solved by Aareyan Manzoor
Log in to reply
SOLUTION TO PROBLEM 2 the polynomial with roots xi is x48+x47+...+x+1=x−1x49−1=0 let y=1−x1 or x=yy−1.then we substitute: (yy−1)49−1=0,y=0 y49=(y−1)49 y48−24y47+376y46−4324y45+.....+491=0 using newtons sum ap1p2p3===s1s1p1−2s2s1p2−s2p1+3s3===24−176−276 we are done.
PROBLEM 8
Let p(x) be a polynomial with integer coefficients. Assume that p(a)=p(b)=p(c)=−1, where a,b,c are three different integers. Prove that p(x) has no integral zeroes.
This problem has been solved by Svatejas Shivakumar
Log in to reply
SOLUTION TO PROBLEM 8
Lemma: If f(x) is a polynomial with integeral coefficients and a is an integeral root of f(x) and m is any integer different from a, then a−m divides f(x).
Proof: On dividing f(x) by x−m we get f(x)=(x−m)q(x)+f(m), where q(x) is a polynomial with integral coefficients. For x=a, we get f(a)=0=(a−m)q(a)+f(m) or f(m)=−(a−m)q(a). Hence a−m divides f(m).
Suppose d is an integeral root of p(x), then by the lemma, d−a,d−b,d−c divides −1 . But −1 has only 2 factors namely −1,1. Hence at least two of a,b,c are the same but a,b,c are different. Hence, p(x) has no integral zeroes.
PROBLEM 9
The polynomial ax3+bx2+cx+d has integral coefficients a,b,c,d with ad odd and bc even. Show that at least one zero of the polynomial is irrational.
Nobody was able to solve this,so Svatejas Shivakumar can post the next problem
Log in to reply
SOLUTION OF PROBLEM 9
Let xi,i=1,2,3 be the rational roots of the given.polynomial. Then (ax)3+b(ax)2+ac(ax)+a2d=0.
Setting y=ax,we get y3+by2+acy+a2d=0.
yi are the three rational zeros of the above equation,i.e. they must be integers. Also, since they are divisors of a2d, they must be odd. Since y1+y2+y3=−b and y1y2+y2y3+y3y1=ac, both b and ac must be odd,i.e., b and c are odd hence bc is odd. This contradicts the assumption that bc is even. Hence at least one zeros of the polynomial is irrational.
PROBLEM 10
The polynomial P(x)=xn+a1xn−1+…+an−1x+1 with nonnegative coefficients a1,…,an−1 has n real roots. Prove that P(2)≥3n.
This problem has been solved by Aditya Agarwal
Log in to reply
Because of the given condition (non-negative coefficients), the roots would be non-positive.
So the polynomial can be expressed as P(x)=(x+b1)...(x+bn) where b_i=-x_i\>0), and \(x_i are the roots.
Now, by the AM-GM inequality, we get, 2+bi≥3bi31
Now by Vieta's Formulas, we have, that the product of the roots is 1.
So P(2)=(2+b1)...(2+bn)≥3n
Those who are saying that the product b1b2...bn=−1, it won't be. Because bi>0, and thus the product has to be greater than −1, so the product would definitely by 1. (The confusion arised because of the substitution, bi=−xi)
PROBLEM 11:
Prove: For any polynomial g(x), deg(g(x))>1, another polynomial k(x) can be substituted for x, in such a way that g(k(x)) can be expressed as a product of non-constant polynomials. (All the polynomials have integral coefficients)
Nobody posted the solution, Aditya Agarwal posted the solution
Log in to reply
Solution for Problem 11:
It is evident that, g(a)−g(x) is divisible by a−x. Now lets us take a, such that a−x, is divisible by g(x), for example, a=k(x)=x+p(x). So, g(k(x)) is divisible by g(x). Now because deg(g(k(x))) is greater than deg(g(x)), the second factor is not constant.
Please post the solution and the next problem as well since no one has solved it within the time limit.
@Aditya Agarwal here is a problem!
Problem 14
Let P(x) be a polynomial of degree n>1 with integer coefficients, and let k be a positive integer. Consider the polynomial Q(x)=P(P(…P(P(x))…)), where P occurs k times. Prove that there are at most n integers t such that Q(t)=t.
Dan Shwarz,Romania
solved by Xuming Liang
Log in to reply
This is the fifth problem of IMO 2006, I recognized it from a documentary about the USA IMO team. I will outline the basic idea of the proof:
The first idea shows why the case k=2 is important(sufficient):
If Q(s)=s but P(s)=s, then P(P(s))=s. In other words, if s is a fixed point of Q but not of P, then it is a fixed point of P(P(x)). So it suffices to consider(count) the fixed points of k=2.
We now prove the case for k=2:
If all fixed points of P(P(x)) are fixed points of P(x), then the result holds because P has at most n fixed points.
If not, then there exist a=b such that P(a)=b,P(b)=a. The next observation is that all pairs (a′,b′) that satisfy the previous equations(a′,b′ need not to be distinct) have the same sum, i.e. a+b=a′+b′=c for some constant c. Thus all the numbers in these pairs(fixed points of P(P(x))) are the roots to the polynomial P(x)+x−c. Since this has the same degree as P(n>1), we are done.
I know this isn't the full solution, so feel free to fill in the steps.
This has totally stumped me! I tried proving it for k=2. But even when I did that, I was unsure of what to do?
Log in to reply
Keep trying though!
The k=2 is a very crucial step!
Problem 17
Consider the sequence of polynomials {Pn(x)}n=1,2,3,... such that Pn(2cosx)=2ncosnx,∀x∈R,∀n∈N∗.
Prove that: 1≤x−2nPn(x)−2≤n,∀x>2,∀n∈N∗
Log in to reply
solution to Problem 17 Pn(2x)=2ncos(narccos(x))=2nTn(x) chebyshev polynomials used here(of the first kind). we know:T2(x)=2x2−1,T3(x)=4x3−3x,Tn+1(x)=2xTn(x)−Tn−1(x) we see hat cases 2,3 are allways satisfied.so 2x−2≤2n+1Tn+1(2x)−2≤(n+1)(2x−2)2x≤2n+12xTn(2x)−Tn−1(2x)≤(n+1)(2x−2)+2xn+1≤4xPn(2x)−4Pn−1(2x)≤((n+1)(2x−2)+2)n+1 we had (2x)n≤Pn(2x)≤(n(2x−2)+2)n−((n−1)(2x−2)+2)n≤−Pn−1(x)≤−(2x)n−1 multiplying and adding 4(2x)n+1−4((n−1)(2x−2)+2)n≤4xPn(2x)−4Pn−1(2x)≤4x(n(2x−2)+2)n−4(2x)n−1 now notice that 4(2x)n+1−4((n−1)(2x−2)+2)n≥xn+14x(n(x−2)+2)n−4(2x)n−1≤((n+1)(2x−2)+2)n+1 Hence proved by induction.
PROBLEM 19
If x1,x2,x3 are the roots of P(x)=x3−9x2+14x−1, find all the possible values of x2x1+x3x2+x1x3.
This problem is solved by both , Xuming and Nihar.
Log in to reply
SOLUTION TO PROBLEM 19:
First I let a=x2x1,b=x3x2,c=x1x3.
Now using x1x2x3=1 (By Vieta's) , we can easily obtain that ab+bc+ac=a1+b1+c1 .
Again by using x1x2x3=1 we obtain :
ab+bc+ca=x13+x23+x33=(x1+x2+x3)3−3(x1+x2+x3)(x1x2+x2x3+x1x3)+3x1x2x3=93−3(9)(14)+3=354
Again by using x1x2x3=1 we obtain : ba+cb+ac=(x1x2)3+(x2x3)3+(x1x3)3. Now we let k=x1x2 , m=x2x3 , n=x1x3 So we have:
k3+m3+n3=(k+m+n)3−3(k+m+n)(km+mn+kn)+3(k2m2n2)
Again by using x1x2x3=1 we obtain : km+mn+kn=x1+x2+x3 and k2m2n2=1. Thus by using Vieta's again ,
k3+m3+n3=(14)3−3(14)(9)+3=2370
We also have (a+b+c)(a1+b1+c1)=3+ba+cb+ac+ab+bc+ca that is :
(a+b+c)(ab+bc+ac)=3+2370+354=2727
We also have (ba+cb+ac)(ab+bc+ca)=a3+b3+c3+a3b3+b3c3+a3c3+3
Now we let p=ab , q=bc ,r=ac , and we have p3+q3+r3=(p+q+r)3−3(p+q+r)(pq+qr+pr)+3p2q2r2 . Now using abc=1 , we have pq+qr+pr=ab+bc+ac ,
p3+q3+r3=(p+q+r)3−3(p+q+r)(pq+qr+pr)+3p2q2r2=(ab+bc+ac)3−3(ab+bc+ac)(a+b+c)+3
and we also have
a3+b3+c3=(a+b+c)3−3(a+b+c)(ab+bc+ac)+3
Adding above both equations ,
p3+q3+r3+a3+b3+c3=(ab+bc+ac)3+(a+b+c)3−6(a+b+c)(ab+bc+ac)+6⇒(ba+cb+ac)(ab+bc+ca)=(ab+bc+ac)3+(a+b+c)3−6(2727)+6⇒(ab+bc+ac)3+(a+b+c)3=855336
Let a+b+c=f , ab+bc+ac=g , we have f3+g3=855336 and fg=2727 and substituting g=f2727 in the former equation and solving the obtained quadratic , we get f=a+b+c=29,94.
HUSH!
Log in to reply
Well done! :D
Log in to reply
@Alan Enrique Ontiveros Salazar
Nice problem !Yeah for sure problem has long calculations ;)
The answers are indeed 94,29. The key to finding the answer is constructing equations using symmetric sums. Note that our desired sum is equal to cyc∑x12x3, which we denote A. Let B denote its symmetric counterpart: cyc∑x12x2.
Note that A+B=sym∑x12x2=9⋅14−3=123.
A⋅B=sym∑x13+sym∑(x11)3+3(x1x2x3)2=2726. Here I omit the routine calculation for the two sums.
Solving for A,B gives us 94,29.
Log in to reply
Yes, that is correct, you used the same method than me.
Wow! I haven't learnt this method...
So who must post the next question? I got the correct answer first but you wrote the solution first...
Log in to reply
Log in to reply
After a very long process , tedious calculations , I got the possible values as approximately 94,29 (It may be wrong too) . Please verify @Alan Enrique Ontiveros Salazar . Thanks.
Problem 21 given a poly F(x)=x4+x+1 if its roots are a,b,c,d find sym∑a10+b91
Log in to reply
Quite a messy problem. Do you have an easier way of doing it? If so, could you please post it since the time is up?
Log in to reply
:p forgot about the contest.
I am not feeling to write the solution..... i will try to later. You can post the next problem(as you reminded me of the contest).
Log in to reply
Log in to reply
Log in to reply
Log in to reply
PROBLEM 12
Let f(x) be a monic polynomial with integral coefficients. If there are four different integers a,b,c,d such that f(a)=f(b)=f(c)=f(d)=5, then show that there is no integer k, so that f(k)=8.
*Solved Svatejas Shivakumar *
Log in to reply
SOLUTION OF PROBLEM 12
f(x)=g(x)(x−a)(x−b)(x−c)(x−d)+5 where g(x) is a polynomial with integral coefficients
If f(k)=8, then g(k)(k−a)(k−b)(k−c)(k−d)=3.
g(k)(k−a)(k−b)(k−c)(k−d) is a product of five integers of which at least four are distinct(since a,b,c,d are distinct) but 3 can be expressed as a product of at most three distinct factors i.e 1×(−1)×(−3).
Hence there is no integer such that f(k)=8.
Log in to reply
Nice solution! Just, edit it to "SOLUTION OF PROBLEM 12". And post problem 13.
PROBLEM 13
The polynomial f(x)=xn+a1xn−1+…+an−1x+an=0 with integral non-zero coefficients, has n real roots. Prove that if the roots are pairwise coprime, then an−1 and an are coprime.
Solved by Aditya Agarwal
Log in to reply
SOLUTION OF PROBLEM 13
Proof by contradiction:
Lets assume that the last two coefficients are not coprime, i.e, gcd(an−1,an)=1.
By Vieta's formulas we have that the product of roots, let them be {zi}1n , is (−1)nan. So this means that one of the roots, let it be z1 is divisible by the number, let it be k, by which an−1 and an are divisible. Also, by Vieta's Formulas, we have z1...zn−1+z1z3...zn+...+z2...zn=(−1)n−1an−1 Now we know that an−1,z1 are divisible by k. So this means that the last term, namely the term which doesn't contain r1, is also divisible by k. But this contradicts the fact that all the roots are coprime. Hence, the last two coefficients should also be coprime.
P.S.: I have not got any good problem at the moment, and also, it doesn't seem that many users are online today. So I would resist posting a problem today, if anyone wants to do it, he can, in place of mine.
Log in to reply
Nice solution! I too don't have any good problem. I searched a lot to find this problem.
Log in to reply
PROBLEM 18 consider P(x)=x4−3x+1=0 has roots xi∀i=1,2,3,4. find cyc∑(x3−3)2x7
Log in to reply
Solution to problem 18
x4=3x−1
x5=3x2−x
x7=3x4−x3=3(3x−1)−x3=−x3+9x−3
x4−3x=−1⟹x3−3=−x1
Then, the sum is cyc∑(−x1)2−x3+9x−3=cyc∑(−x5+9x3−3x2)=cyc∑=(−(3x2−x)+9x3−3x2)=cyc∑(9x3−6x2+x)
We know that S1=0,S2=0,S3=3,S4=1 and, by Newton's Sums, P1=0,P2=0,P3=9. So the sum is 9(9)−6(0)+0=81.
Log in to reply
Correct! Post next problem!